Online
axil Verified User
  • Member since Oct 10th 2014
  • Last Activity:

Posts by axil

    The SPP generated chiral magnetic flux tube performs two basic LENR functions: 1 - destabilizes the nucleon, and 2 - stabilize any nuclear activation that the first mentioned function produces.

    Yes. So what?

    (Hint: "Therefore, this supports my LENR theory" is a non-sequitur.)


    One negative argument is that the quasiparticles do not manifest chiral spin. Post #2,464 was presented to forestall that argument.

    Well, right off the bat you started running in the wrong direction. So I really can't help you until you decide to stop and turn around.


    BTW, you ignored my basic questions above.


    How about describing the involvement of orbital vs spin angular momentum (or both) in your theory?


    Chirality is not a intrinsic property of orbital vs spin angular momentum. It is a property of particle spin including quarks and quasiparticles. Particle spin is the source of magnetism. This magnetism is what is the common interface between the SPP and the quark.


    I am doing my best to layout an all encompassing LENR theory based on the SPP. I am not discouraged by the censorship that I am experiencing lately or the negativity from learned experts that could greatly help in perfecting the theory.


    I addressed THHuxley references. They all concerned light matter interaction. Light matter interactions boils down to polariton formation and behavior.


    You have given the laser based U232 stabilization experiment short shrift. Why?


    The Purcell effect


    A polariton is not a photon. The Polariton is a quasiparticle and if a quasiparticle has a spin then it is chiral. All spin is chiral.


    You missed this reference


    https://www.nature.com/articles/524008b

    Particle physics: Only left-handed particles decay


    Read up on chirality. From what you are saying, I don't think you have seen this in your education.


    https://en.wikipedia.org/wiki/Chirality_(physics)


    Only left-handed fermions and right-handed antifermions interact with the weak interaction.



    Axil, THH & others,


    According to Holmlid, ultra-dense deuterium decays via the weak force in mesons. What is surprising here is less the occurence of a weak decay - given the length and time scales at play - than the absence of the usual D-D fusion and decay via the fast strong interaction.


    Does anyone know a process that could prevent the decay of an excited D-D via the strong force?


    See


    Center Vortex Flux Tube Interference.


    The Surface Plasmon Polariton (SPP) generates a magnetic flux tube that destabilizes the quark flux tube interconnections in the nucleon. This destabilization is what generates mesons.


    The SPP form in the spin wave that cover the surface of the ultra dense material.


    The citation supporting my statement was included in these threads


    Nezt...


    How are the flux tubes produced


    The Surface Plasmon Polaritons generate two counter rotating spin currents that are chiral polarized. One current is left handed spin polarized and the other current is right handed spin polarized. These spin currents generate a falaco soliton. The spinning flux tubes connect these two rotating spin currents. Notice in the picture below that two counter rotating spin flows are represented.


    Pairs-of-Falaco-solitons-An-example-of-solid-2-dimensional-0-surgery.jpg


    I need to stop now because I will cause these posts to be sent to the clearance thread.

    Except that, as usual, there is no peer-reviewed manuscript providing evidence to support Axil's incoherent (but very "scientific" and intelligent sounding) babbling.


    Or go ahead Axil, please enlighten us all and prove me wrong by providing the peer-reviewed manuscript supporting your statement that the "cause of LENR is left handed chiral polarized magnetic flux tubes that instantly stabilize radioactive isotopes" supported by experiment.


    First look at this experiment


    External Content www.youtube.com
    Content embedded from external sources will not be displayed without your consent.
    Through the activation of external content, you agree that personal data may be transferred to third party platforms. We have provided more information on this in our privacy policy.


    Can...


    Now you can tell Alan Smith how Rossi make gamma radiation go away so that Alan can produce excess heat instead of gammas.


    Can, while you are at it, you might explain how Alan Smith's experiment is producing gammas and yet is not producing any radioactive activation of the reactor.

    axil : It's time to move off. Alan claims nothing! It is all your phantasy!


    If you believe that the standard model explains LENR, then go to a classic physics forum...


    The cause of LENR is left handed chiral polarized magnetic flux tubes that instantly stabilize radioactive isotopes. This theory is supported by experiment.

    Does Rossi read LF?


    For those who wish both Rossi and R. Mills ill, be joyful in that these plasma based LENR reactors will be found to produce muons by the boatload. Rossi and Mills together with all their investors will be ruined when this muon exposure problem is detected.